LSAT and Law School Admissions Forum

Get expert LSAT preparation and law school admissions advice from PowerScore Test Preparation.

 Administrator
PowerScore Staff
  • PowerScore Staff
  • Posts: 8916
  • Joined: Feb 02, 2011
|
#40559
Complete Question Explanation
(The complete setup for this game can be found here: lsat/viewtopic.php?t=15423)

The correct answer choice is (D)

The question stem creates a WX block. Because Z :longline: X from the second rule, this WX block must appear in the last three slots, leading to the following basic diagram for this question:
PT70 -Game_#1_#6_diagram 1.png
Answer choice (A): This answer choice is incorrect because if U performs fifth, then there would be no acceptable positioning for the WX block.

Answer choice (B): This answer choice is incorrect because V must perform first or second.

Answer choice (C): This answer choice is incorrect because W must perform fourth or fifth.

Answer choice (D): This is the correct answer choice.

Answer choice (E): This answer choice is incorrect because Z must perform second or third.

Alternatively, if you did not have the group information about the first three and last three slots, you would approach this question by focusing on the sequential relationships. The question stem produces two sequences:
PT70 -Game_#1_#6_diagram 2.png
When combined, these two sequences allow you to answer the question as well, albeit a bit more slowly.
You do not have the required permissions to view the files attached to this post.

Get the most out of your LSAT Prep Plus subscription.

Analyze and track your performance with our Testing and Analytics Package.